Learning Feynman Diagrams from Peskin & Schroeder

In summary, the conversation discusses the use of Feynman diagrams and constant factors in Peskin and Schroeder's book. It specifically focuses on equation 4.44 and 4.45 and the factors involved in their calculation. The conversation also mentions the use of symmetry factors and how they can affect the overall result of the calculation. The conclusion is that by dividing by the symmetry factor, the correct result can be obtained.
  • #1
Icosahedron
54
0

Homework Statement


I’m trying to learn Feynman diagrams from Peskin and Schroeder. I’m stuck right now on page 91-93, especially about constant factors and symmetry factors. Equation 4.44 and the two constants 3 and 12 in it make perfect sense to me. The trouble starts with 4.45. I understand it is one possible contraction from 10368 that all give the same expression.

Why the 1/3! in 4.45?

To get similar result for 4.45 as for 4.44, we need to replace the 1/3! With 10368 and add other expressions each with an overall constants. Correct?

On page 93 they say (first paragraph, second to last line) adding at each vertex this integral times minus I times game gives the overall constant. How on Earth does that give 10368?

What constant is why by a symmetry constant 8 to large?



Homework Equations


Peskin p.91-93


The Attempt at a Solution



see above
 
Physics news on Phys.org
  • #2
Icosahedron said:

Homework Statement


I’m trying to learn Feynman diagrams from Peskin and Schroeder. I’m stuck right now on page 91-93, especially about constant factors and symmetry factors. Equation 4.44 and the two constants 3 and 12 in it make perfect sense to me. The trouble starts with 4.45. I understand it is one possible contraction from 10368 that all give the same expression.

Why the 1/3! in 4.45?
this is the 1/n! coming from the fact that you are expanding an exponential!
To get similar result for 4.45 as for 4.44, we need to replace the 1/3! With 10368 and add other expressions each with an overall constants. Correct?

On page 93 they say (first paragraph, second to last line) adding at each vertex this integral times minus I times game gives the overall constant.

I don't see this in my edition...and beside, the sentence does not make sense...
How on Earth does that give 10368?

What constant is why by a symmetry constant 8 to large?


They are saying the following: the product of all the combinatoric factors almost exactly cancels out the 1/3! (from the exponential expansion) and the powers of 1/4! (from the Feynman rule for the vertex. So let's say you would simply forget all those factors altogether and use an overall factor of one exactly. You would almost get it right except that your factor would be 8 times too large. If you use one and divide by the symmetry factor you will get the correct answer.

Hope this clarifies things.

Patrick
 
  • #3
Hi nrqed,
many thanks for your reply, but unfortunately it is still unclear

the sentence does not make sense...
Ooops, I should not drink beer while formulating posts at PF. Anyway, they say in the first paragraph in the second to last line: "it is therefore conventional to associate the expression 'integral times -i times gamma' with each vertex".

Now again my question: what factor do I get by applying this scheme? 10368 times 8? How?

Also again, to get a similar result for 4.45 as for 4.44, do we need to replace the 1/3! with 10368 and add other expressions each with an overall constants just as we did in 4.44?

you say
the product of all the combinatoric factors almost exactly cancels out the 1/3! (from the exponential expansion) and the powers of 1/4!

I don't see that. 10368 cancels out 1/3! times (1/4!)^3??

thank you
 
  • #4
Icosahedron said:
Hi nrqed,
many thanks for your reply, but unfortunately it is still unclear


Ooops, I should not drink beer while formulating posts at PF. Anyway, they say in the first paragraph in the second to last line: "it is therefore conventional to associate the expression 'integral times -i times gamma' with each vertex".

Now again my question: what factor do I get by applying this scheme? 10368 times 8? How?

Also again, to get a similar result for 4.45 as for 4.44, do we need to replace the 1/3! with 10368 and add other expressions each with an overall constants just as we did in 4.44?

you say


I don't see that. 10368 cancels out 1/3! times (1/4!)^3??

thank you

I said almost cancels out!

If you do the full calculation, you have a result of the following form:

1/3! (for the expansion of the exponential) times (1/4!)^3 for the vertices times a messy combinatorial calculation which comes out to be equal to 10368.

So you get a huge number divided by a huge number and the result is 10368/(3! (4!)^3) =1/8.

They are saying that the huge number coming from the combinatorial is almost equal to the huge number coming from the factorials due to the vertex rule and the exponential expansion.

So they say: could we simply forget about all the combinatorial stuff, forget about the factorial coming from the exponential expnasion and simply use a vertex rule lambda instead of lambda/4! ? In that case, you would just have an overall factor of ONE.

But, obviously, you can't do that because the combinatorial stuff does not exactly cancel out the product of factorials.

So should we go back to doing the full calculation? No. A shortcut is the following:

Forget about the 1/n! coming from the exponential expansion. Do not include a factor of 1/4! in the vertex rule. Do not do all the combinatorial stuff. BUT you must divide by the symmetry factor. This will give the correct result in the end.
 
  • #5
thanks nrqed! Clear now.

I will now try to accustom myself with symmetry factors, so maybe I have to come back and ask some more.

but for now, many thanks
 
  • #6
Icosahedron said:
thanks nrqed! Clear now.

I will now try to accustom myself with symmetry factors, so maybe I have to come back and ask some more.

but for now, many thanks

You are very welcome:cool:
 

Related to Learning Feynman Diagrams from Peskin & Schroeder

1. What are Feynman diagrams and why are they important in particle physics?

Feynman diagrams are graphical representations of mathematical expressions that describe interactions between elementary particles. They are important in particle physics because they provide a visual tool for understanding complex particle interactions and calculating their probabilities.

2. What is the best way to learn Feynman diagrams from Peskin & Schroeder?

The best way to learn Feynman diagrams from Peskin & Schroeder is to start with the basics, such as understanding the symbols and conventions used in the diagrams, and then gradually progress to more advanced concepts. It is also helpful to practice drawing and interpreting diagrams for different particle interactions.

3. Do I need a strong background in mathematics to understand Feynman diagrams?

While a basic understanding of mathematics is necessary to fully comprehend Feynman diagrams, it is not necessary to have a strong background in mathematics to understand the basic concepts and use them for calculations. A familiarity with algebra, trigonometry, and calculus is sufficient.

4. Can Feynman diagrams be used for any type of particle interaction?

Yes, Feynman diagrams can be used to describe any type of particle interaction, including electromagnetic, weak, and strong interactions. They can also be used for calculations in quantum field theory and high-energy physics.

5. Are there any resources or tools that can help with learning Feynman diagrams from Peskin & Schroeder?

Yes, there are several resources and tools available to help with learning Feynman diagrams from Peskin & Schroeder. These include online tutorials, interactive simulations, and study guides. It is also helpful to work through practice problems and seek guidance from a mentor or instructor.

Similar threads

  • Advanced Physics Homework Help
Replies
3
Views
2K
  • Advanced Physics Homework Help
Replies
4
Views
2K
  • Advanced Physics Homework Help
Replies
1
Views
873
  • Advanced Physics Homework Help
Replies
1
Views
2K
  • Quantum Physics
Replies
6
Views
3K
  • Advanced Physics Homework Help
Replies
1
Views
5K
  • Advanced Physics Homework Help
Replies
2
Views
2K
  • High Energy, Nuclear, Particle Physics
Replies
8
Views
2K
  • High Energy, Nuclear, Particle Physics
Replies
4
Views
1K
  • High Energy, Nuclear, Particle Physics
Replies
3
Views
2K
Back
Top